LSAT and Law School Admissions Forum

Get expert LSAT preparation and law school admissions advice from PowerScore Test Preparation.

User avatar
 Ryan Twomey
PowerScore Staff
  • PowerScore Staff
  • Posts: 141
  • Joined: Mar 04, 2021
|
#86423
Hey Cornflakes,

I like the explanation you gave and I think it is great. With a resolve the paradox correct answer choice, we are looking to add new information to explain an outcome. In this case our outcome is essentially 'we will still have blackouts even if people turn off air conditioning in their homes.'

Answer choice A tells us that there are significant drains on the system from other sources, which I would react to by saying 'well of course there are, so what?' That does not explain why the blackout occurs in the heatwave.

Whereas B perfectly explains our outcome. If most of the air conditioning is used to cool businesses and factories, that makes the home air conditioning request less relevant and explains why we will still face blackouts.

Hope this helps. Great line of thinking there.

Best,
Ryan
User avatar
 CJ12345:
  • Posts: 56
  • Joined: May 25, 2023
|
#104609
Hi, Powerscore,
based on the previous discussion, I understand B is preferable to A. However, I still have questions about A.
The stimulus is saying air conditioning -->(cause) blackout. Then, it said when the cause gets reduced, the effect does not reduce. A is saying that it is possible that other things also cause blackouts. Thus, even if air conditioning is reduced, the blackout could still not be reduced (maybe the other thing is so significant that kind of counterbalances any effect reduction in air conditioning has on blackout?). Why is this line of reasoning not correct? Also, if we compare this Q to PT32S1Q10, this PT32 question's correct AC points out a similar reasoning to A. I got very confused as to why in this question, this kind of reasoning is correct, but not correct in PT63 Q.
 Robert Carroll
PowerScore Staff
  • PowerScore Staff
  • Posts: 1787
  • Joined: Dec 06, 2013
|
#104665
CJ12345:,

Answer choice (A) doesn't say what you claimed: "A is saying that it is possible that other things also cause blackouts." It's just saying that other things contribute to draining the electrical system. There's no indication in answer choice (A) that those things are enough of a drain to contribute to blackouts, though the stimulus tells us that AC use IS severe enough to contribute to blackouts. So answer choice (A) is not introducing another cause of blackouts, which is what it needs to do to be correct.

PT32S1Q10 is a Flaw question. I don't see the relevance to this question, which is a Resolve the Paradox.

Robert Carroll

Get the most out of your LSAT Prep Plus subscription.

Analyze and track your performance with our Testing and Analytics Package.